A geometry problem by Ankit Kumar Jain

Geometry Level 4

cos ( π 7 ) + cos ( 3 π 7 ) + cos ( 5 π 7 ) = ? \large \cos \left(\frac \pi 7\right) + \cos \left(\frac {3\pi}{7}\right) + \cos \left(\frac{5\pi}{7}\right) = \, ?

This is part of the set My Problems and THRILLER


The answer is 0.5.

This section requires Javascript.
You are seeing this because something didn't load right. We suggest you, (a) try refreshing the page, (b) enabling javascript if it is disabled on your browser and, finally, (c) loading the non-javascript version of this page . We're sorry about the hassle.

3 solutions

Ankit Kumar Jain
Feb 15, 2017

Let z = c o s ( π 7 ) + i s i n ( π 7 ) z = cos\left(\dfrac{\pi}{7}\right) + isin\left(\dfrac{\pi}{7}\right) .

By de moivre's theorem , z 7 = c o s ( π ) + i s i n ( π ) = 1 z^7 = cos\left(\pi\right) + isin\left(\pi\right) = -1

z 7 + 1 = 0 \Rightarrow z^7 + 1 = 0

( z + 1 ) ( z 6 z 5 + z 4 z 3 + z 2 z + 1 ) = 0 \Rightarrow \left(z + 1\right)\left(z^6 - z^5 + z^4 - z^3 + z^2 - z +1\right) = 0

But since the first factor 0 \neq 0 , therefore the second factor = 0 = 0 .


By de movier's theorem,

c o s ( π 7 ) + c o s ( 3 π 7 ) + c o s ( 5 π 7 ) cos\left(\dfrac{\pi}{7}\right) + cos\left(\dfrac{3\pi}{7}\right) + cos\left(\dfrac{5\pi}{7}\right)

= 1 2 ( ( z + 1 z ) + ( z 3 + 1 z 3 ) + ( z 5 + 1 z 5 ) ) = \dfrac{1}{2} \left( \left( z + \dfrac{1}{z}\right) + \left(z^3 + \dfrac{1}{z^3}\right) + \left(z^5 + \dfrac{1}{z^5} \right) \right)

= 1 2 ( z 6 + z 4 + z 8 + z 2 + z 10 + 1 z 5 ) = \dfrac{1}{2} \left( \dfrac{z^6 + z^4 + z^8 + z^2 + z^{10} + 1}{z^5} \right) .

= 1 2 ( z 6 + z 4 + z 8 + z 2 + z 10 + 1 z 5 ) + 1 2 1 2 = \dfrac{1}{2} \left( \dfrac{z^6 + z^4 + z^8 + z^2 + z^{10} + 1}{z^5} \right) + \dfrac{1}{2} - \dfrac{1}{2} .

Using z 7 = 1 z^7 = -1 to simplify the expression gives ,

1 2 ( z 6 z 5 + z 4 z 3 + z 2 z + 1 z 5 ) + 1 2 \dfrac{1}{2}\left(\dfrac{z^6 - z^5 + z^4 - z^3 + z^2 - z + 1}{z^5}\right) + \dfrac{1}{2}

Using , ( z 6 z 5 + z 4 z 3 + z 2 z + 1 ) = 0 \left(z^6 - z^5 + z^4 - z^3 + z^2 - z +1\right) = 0 .

We get the answer as 1 2 \boxed{\dfrac{1}{2}} .

Chew-Seong Cheong
Feb 15, 2017

In general, k = 0 n 1 cos ( 2 k + 1 2 n + 1 π ) = 1 2 \displaystyle \sum_{k=0}^{n-1} \cos \left(\frac {2k+1}{2n+1}\pi \right) = \frac 12 , consider the following.

S = k = 0 n 1 cos ( 2 k + 1 2 n + 1 π ) = k = 0 n 1 { e 2 k + 1 2 n + 1 π i } where { z } is the real part of complex number z = { k = 0 n 1 e 2 k + 1 2 n + 1 π i } = { e π i 2 n + 1 k = 0 n 1 e 2 k π i 2 n + 1 } = { e π i 2 n + 1 ( 1 e 2 n π i 2 n + 1 1 e 2 π i 2 n + 1 ) } = { e π i 2 n + 1 + 1 1 e 2 π i 2 n + 1 } = { 1 1 e π i 2 n + 1 } = { 1 1 cos π 2 n + 1 i sin π 2 n + 1 } = { 1 cos π 2 n + 1 + i sin π 2 n + 1 ( 1 cos π 2 n + 1 ) 2 + sin 2 π 2 n + 1 } = { 1 cos π 2 n + 1 + i sin π 2 n + 1 2 2 cos π 2 n + 1 } = 1 2 \begin{aligned} S & = \sum_{k=0}^{n-1} \cos \left(\frac {2k+1}{2n+1}\pi \right) \\ & = \sum_{k=0}^{n-1} \Re \left \{ e^{\frac {2k+1}{2n+ 1} \pi i} \right \} & \small \color{#3D99F6} \text{where } \Re \{ z \} \text{ is the real part of complex number }z \\ & = \Re \left \{\sum_{k=0}^{n-1} e^{\frac {2k+1}{2n+1}\pi i} \right \} \\ & = \Re \left \{e^{\frac {\pi i}{2n+1}} \sum_{k=0}^{n-1} e^{\frac {2k \pi i}{2n+1}} \right \} \\ & = \Re \left \{e^{\frac {\pi i}{2n+1}} \left( \frac {1 - e^{\frac {2n \pi i}{2n+1}}}{1 - e^{\frac {2\pi i}{2n+1}}} \right) \right \} \\ & = \Re \left \{\frac {e^{\frac {\pi i}{2n+1}} + 1}{1 - e^{\frac {2\pi i}{2n+1}}} \right \} \\ & = \Re \left \{\frac 1{1 - e^{\frac {\pi i}{2n+1}}} \right \} \\ & = \Re \left \{\frac 1{1 - \cos {\frac \pi{2n+1}} - i \sin {\frac \pi{2n+1}}} \right \} \\ & = \Re \left \{\frac {1 - \cos {\frac \pi{2n+1}} + i \sin {\frac \pi{2n+1}}}{\left(1 - \cos {\frac \pi{2n+1}}\right)^2 + \sin^2 {\frac \pi{2n+1}}} \right \} \\ & = \Re \left \{\frac {1 - \cos {\frac \pi{2n+1}} + i \sin {\frac \pi{2n+1}}}{2 - 2\cos {\frac \pi{2n+1}}} \right \} \\ & = \boxed{\dfrac 12} \end{aligned}

Great solution sir!!

Ankit Kumar Jain - 4 years, 3 months ago
Prakhar Bindal
Feb 15, 2017

Directly use the formula of summation of cosines when the angles are in AP .

To get 1/2

Can you please share the formula and its proof because I am not aware of it?

Ankit Kumar Jain - 4 years, 3 months ago

Log in to reply

http://math.stackexchange.com/questions/117114/sum-cos-when-angles-are-in-arithmetic-progression

Prakhar Bindal - 4 years, 3 months ago

Log in to reply

@Prakhar Bindal Thanks!

Ankit Kumar Jain - 4 years, 3 months ago

0 pending reports

×

Problem Loading...

Note Loading...

Set Loading...